0% found this document useful (0 votes)
940 views11 pages

2021 AMC 12A: 1 Answer Key

This document contains the answer key and solutions to problems on the 2021 AMC 12A exam. It lists the correct answers to 25 multiple choice problems in the first section. The second section provides the problems, solutions, and reasoning for problems 1 through 13. The summary focuses on the high-level purpose and structure of the document. 1. This document contains an answer key and solutions to problems from the 2021 AMC 12A exam. 2. The first section lists the correct answers to 25 multiple choice problems. 3. The second section presents 13 math problems along with their solutions and reasoning. Each problem solution is 1-2 sentences explaining the reasoning.

Uploaded by

chandru
Copyright
© © All Rights Reserved
We take content rights seriously. If you suspect this is your content, claim it here.
Available Formats
Download as PDF, TXT or read online on Scribd
0% found this document useful (0 votes)
940 views11 pages

2021 AMC 12A: 1 Answer Key

This document contains the answer key and solutions to problems on the 2021 AMC 12A exam. It lists the correct answers to 25 multiple choice problems in the first section. The second section provides the problems, solutions, and reasoning for problems 1 through 13. The summary focuses on the high-level purpose and structure of the document. 1. This document contains an answer key and solutions to problems from the 2021 AMC 12A exam. 2. The first section lists the correct answers to 25 multiple choice problems. 3. The second section presents 13 math problems along with their solutions and reasoning. Each problem solution is 1-2 sentences explaining the reasoning.

Uploaded by

chandru
Copyright
© © All Rights Reserved
We take content rights seriously. If you suspect this is your content, claim it here.
Available Formats
Download as PDF, TXT or read online on Scribd
You are on page 1/ 11

2021 AMC 12A

Peeyush Pandaya et al.

February 2021

1 Answer Key
1. B 6. C 11. C 16. C 21. A

2. D 7. D 12. A 17. D 22. D

3. D 8. C 13. B 18. E 23. D

4. D 9. C 14. E 19. C 24. D

5. E 10. E 15. D 20. B 25. E

2 Problems and Solutions


Problem 1. What is the value of

21+2+3 − (21 + 22 + 23 )?

(A) 0 (B) 50 (C) 52 (D) 54 (E) 57

Solution. 26 − (21 + 22 + 23 ) = 64 − (2 + 4 + 8) = 64 − 14 = (B) 50 



Problem 2. Under what conditions is a2 + b2 = a + b true, where a and b are real numbers?
(A) It is never true
(B) It is true if and only if ab = 0
(C) It is true if and only if a + b ≥ 0
(D) It is true if and only if ab = 0 and a + b ≥ 0
(E) It is always true

Solution. It is clear that both sides of the equation must be nonnegative. Squaring,

a2 + b2 = a2 + 2ab + b2 =⇒ ab = 0

The answer is (D) 

1
2021 AMC 12A Solution Manual

Problem 3. The sum of two natural numbers is 17, 402. One of the two numbers is divisible by
10. If the units digit of that number is erased, the other number is obtained. What is the difference
of these two numbers?
(A) 10, 272 (B) 11, 700 (C) 13, 362 (D) 14, 238 (E) 15, 426

Solution. Let the first number mentioned be 10n; the second is n. Then 10n + n = 17, 402, from
9
which it follows that 10n − n = 11 · 17, 402 = (D) 14, 238 

Problem 4. Tom has a collection of 13 snakes, 4 of which are purple and 5 of which are happy.
He observes that

• all of his happy snakes can add,


• none of his purple snakes can subtract
• all of his snakes that can’t subtract also can’t add.

Which of these conclusions can be drawn about Tom’s snakes?

Solution. Together, the second and third conditions imply that none of Tom’s purple snakes can
add. Thus, (D) is correct: happy snakes are not purple. 

Problem 5. When a student multiplied the number 66 by the repeating decimal,


1.a b a b . . . = 1.a b,
where a and b are digits, he did not notice the notation and just multiplied 66 times 1.a b. Later
he found that his answer is 0.5 less than the correct answer. What is the 2-digit integer a b?
(A) 15 (B) 30 (C) 45 (D) 60 (E) 75
   
ab ab
Solution. The student computed 66 1 + 100 ; the correct answer is 66 1 + 99 . Thus,
 
1 1
66 · (a b) · − = 0.5 =⇒ a b = (E) 75
99 100


Problem 6. A deck of cards has only red cards and black cards. The probability of a randomly
chosen card being red is 13 . When 4 black cards are added to the deck, the probability of choosing
red becomes 14 . How many cards were in the deck originally?
(A) 6 (B) 9 (C) 12 (D) 15 (E) 18

Solution. If the deck begins with x red cards and 3x cards in total, then
x 1
= =⇒ x = 4 =⇒ 3x = (C) 12
3x + 4 4


Problem 7. What is the least possible value of (xy − 1)2 + (x + y)2 for real numbers x and y?

Solution. We have (xy − 1)2 + (x + y)2 = (xy)2 − 2xy + 1 + x2 + 2xy + y 2 = x2 y 2 + x2 + y 2 + 1 =


(x2 + 1)(y 2 + 1), which achieves a minimum of (D) 1 at x = y = 0. 

2
2021 AMC 12A Solution Manual

Problem 8. A sequence of numbers is defined by D0 = 0, D1 = 0, D2 = 1, and Dn = Dn−1 + Dn−3


for n ≥ 3. What are the parities (evenness or oddness) of the triple of numbers (D2021 , D2022 , D2023 ),
where E denotes even and O denotes odd?

Solution.
Di 0/1
D0 0
D1 0
D2 1
D3 D2 + D0 = 1 + 0 = 1
D4 D3 + D1 = 1 + 0 = 1
D5 D4 + D2 = 1 + 1 = 0
D6 D5 + D3 = 0 + 1 = 1
D7 D6 + D4 = 1 + 1 = 0
D8 D7 + D5 = 0 + 0 = 0
D9 D8 + D6 = 0 + 1 = 1
D10 D9 + D7 = 1 + 0 = 1
.. ..
. .
We can see that the pattern repeats in cycles of length 7, and as 2021 ≡ 5 mod 7, we have D2021 =
D5 , D2022 = D6 , D2023 = D7 =⇒ (C) (E, O, E) 

Problem 9. Which of the following is equivalent to

(2 + 3)(22 + 32 )(24 + 34 )(28 + 38 )(216 + 316 )(232 + 332 )(264 + 364 )?

(A) 3127 +2127 (B) 3127 +2127 +2·363 +3·263 (C) 3128 −2128 (D) 3128 +2128 (E) 5127

Solution.

(3 − 2)(2 + 3)(22 + 32 )(24 + 34 )(28 + 38 )(216 + 316 )(232 + 332 )(264 + 364 )
= (32 − 22 )(22 + 32 )(24 + 34 )(28 + 38 )(216 + 316 )(232 + 332 )(264 + 364 )
= (34 − 24 )(28 + 38 )(216 + 316 )(232 + 332 )(264 + 364 )
..
.
= (364 − 264 )(264 + 364 )
= (C) 3128 − 2128

3
2021 AMC 12A Solution Manual

Problem 10. Two right circular cones with vertices facing down as shown in the figure below
contain the same amount of liquid. The radii of the tops of the liquid surfaces are 3cm and 6cm.
Into each cone is dropped a spherical marble of radius 1cm, which sinks to the bottom and is
completely submerged without spilling any liquid. What is the ratio of the rise of the liquid level
the narrow cone to the rise of the liquid level in the wide cone?

(A) 1 : 1 (B) 47 : 43 (C) 2 : 1 (D) 40 : 13 (E) 4 : 1


2
Solution. The two cones have equal volume, so the height of the first is 36 = 4 times that of the
second. Since the volumes increase by equal proportions, the heights increase by equal proportions.
Thus, the ratio of the rise in liquid levels is (E) 4 : 1 

Problem 11. A laser is placed at the point (3, 5). The laser beam travels in a straight line. Larry
wants the beam to hit and bounce off the y-axis, then hit and bounce off the x-axis, then hit the
point (7, 5). What is the total distance the beam will travel along this path?
√ √ √ √ √
(A) 2 10 (B) 5 2 (C) 10 2 (D) 15 2 (E) 10 5

Solution. Reflect about the y-axis then the x-axis. It is well-known that the image under the two
reflections must be a straight line.

p √
The answer is (3 − (−7))2 + (5 − (−5))2 = (C) 10 2 

4
2021 AMC 12A Solution Manual

Problem 12. All the roots of polynomial z 6 − 10z 5 + Az 4 + Bz 3 + Cz 2 + Dz + 16 are positive


integers, possibly repeated. What is the value of B?
(A) − 88 (B) − 80 (C) − 64 (D) − 41 (E) − 40

Solution. By Vieta’s, the sum of the 6 roots is 10 and the product is 16, hence they are all
powers of 2. It is not hard to find that the only working unordered sextuple is (2, 2, 2, 2, 1, 1).
As (z − 2)4 = z 4 − 8z 3 + 24z 2 − 32z + 16 and (z − 1)2 = z 2 − 2z + 1, the z 3 coefficient is
−8 · 1 + 24 · (−2) + (−32) · 1 = (B) − 88 

Problem 13. Of the following complex numbers z, which has the property that z 5 has the greatest
real part?
√ √ √ √
(A) − 2 (B) − 3 + i (C) − 2 + 2i (D) − 1 + 3i (E) 2i

Solution. The magnitude of each complex number is the same, so it suffices to look at the argu-
ment. The angles are π, 5π 3π π π π 15π 5π
6 , 4 , 3 , and 2 , which after raising to the 5th power give π, 6 , 4 , 3 ,
and 2 . We seek the angle that reaches farthest to the right (smallest argument), which is π6 . Thus,
π

our answer is (B) − 3 + i


Problem 14. What is the value of


20 100
! !
k2
X X
log5k 3 · log9k 25k ?
k=1 k=1

(A) 21 (B) 100 log5 3 (C) 200 log3 5 (D) 2, 200 (E) 21, 000

Solution.
20 20 20 20
X 2
X  k X   X 20 · 21
log5k 3k = log5k 3k = k log5k 3k = k log5 3 = log5 3 = 210 log5 3
2
k=1 k=1 k=1 k=1

And,

100
X 100
X 100
X
log9k 25k = log32k 52k = log3 5 = 100 log3 5
k=1 k=1 k=1

Therefore, their product is 210 log5 3 · 100 log3 5 = (E) 21, 000 

5
2021 AMC 12A Solution Manual

Problem 15. A choir director must select a group of singers from among his 6 tenors and 8 basses.
The only requirements are that the difference between the numbers of tenors and basses must be a
multiple of 4, and the group must have at least one singer. Let be the number of groups that could
be selected. What is the remainder when N is divided by 100?
(A) 47 (B) 48 (C) 83 (D) 95 (E) 96

Solution. Suppose we mark down (1) the tenors that are in the group, and (2) the basses that
aren’t in the group. Then we necessarily mark down a number of people that is a multiple of 4.
This is also sufficient; we mark down some people numbering a multiple of 4, then select the marked
tenors and unmarked basses to form our choir. Clearly, we just mark down at least one person.
The answer is thus
           
14 14 14 1 14 14 14
+ + = + + ··· + − 1 = 40 95
4 8 12 2 0 2 14

The desired answer is (D) 95




Problem 16. In the following list of numbers, the integer n appears times in the list for 1 ≤ n ≤
200.
1, 2, 2, 3, 3, 3, 4, 4, 4, 4, . . . , 200, 200, . . . 200.

What is the median of the numbers in this list?


(A) 100.5 (B) 134 (C) 142 (D) 150.5 (E) 167

Solution. For general n, we have n(n+1)2 numbers. We want to approximate a such that a(a+1)
2 is
n(n+1)
close to 4 . Since the formula is a quadratic in n and we are halving
√ this value, we can find
that a is approximately √12 n. Plugging in n = 200, this is about 100 2, or 141. Of the answer
choices, (C) 142 is the closest, and indeed it is our answer.
141·142 200·201
To verify, we can see that 2 = 10011 and 2 = 20100, so clearly 142 works. 

Problem 17. Trapezoid ABCD has AB k CD, BC = CD = 43, and AD ⊥ BD. Let O be the
intersection of the diagonals AC and BD, and let P be the midpoint of BD. Given that OP = 11,

the length AD can be written in the form m n, where m and n are positive integers and n is not
divisible by the square of any prime. What is m + n?
(A) 65 (B) 132 (C) 157 (D) 194 (E) 215

Solution. Let M be the intersection of CP and AB. Since DCBM is a kite, and CM ⊥ BD, we
have M P ⊥ P B, and by considering the homothethy taking 4M BD to 4ABD with scale factor
2, we can see that M is the midpoint of AB. In particular, we have M P = 12 AD.
Since AD ⊥ BD, we have AD ⊥ DO and thus ∠ADO = 90◦ , and as CD = CB, we have CP ⊥ BD
and ∠CP D = ∠CP O = 90◦ . Also, ∠AOD = ∠COP , so 4AOD ∼ 4COP . Therefore,
DO DO AD AD
= = = = 2,
11 PO CP MP
√ √ √
so DO = 22. Thus, AD = AB 2 − BD2 = 862 − 662 = 4 190 =⇒ m + n = (D) 194 

6
2021 AMC 12A Solution Manual

Problem 18. Let f be a function defined on the set of positive rational numbers with the property
that f (a · b) = f (a) + f (b) for all positive rational numbers a and b. Suppose that f also has the
property that f (p) = p for every prime number p. For which of the following numbers x is f (x) < 0?
17 11 7 7 25
(A) 32 (B) 16 (C) 9 (D) 6 (E) 11

Solution. Note that f (a · 1) = f (a) + f (1) =⇒ f (1) = 0, and


     
1 1 1
0 = f (1) = f p · = f (p) + f =⇒ f = −p.
p p p

In particular, it follows by induction that f (pk ) = kp for each k ∈ Z. Thus,

(A) f (2−5 · 17) = −5 · 2 + 17 = 7


(B) f (2−4 · 11) = −4 · 2 + 11 = 3
(C) f (3−2 · 7) = −2 · 3 + 7 = 1
(D) f (2−1 · 3−1 · 7) = −1 · 2 + (−1) · 3 + 7 = 2
(E) f (52 · 11−1 ) = 2 · 5 − 11 = −1

25
The answer is (E) 
11
π π
 
Problem 19. How many solutions does the equation sin 2 cos(x) = cos 2 sin(x) have in the
closed interval [0, π]?
(A) 0 (B) 1 (C) 2 (D) 3 (E) 4

Solution. Note on the interval ( π2 , π], the left-hand side is negative while the right-hand side is
positive. We thus restrict our attention to [0, π2 ]. The arguments A = π2 cos x and B = π2 sin x are
both between 0 and π2 . For sin A = cos B in A, B ∈ [0, π2 ], we must have A + B = π2 . This implies
sin x + cos x = 1, hence x = 0, π2 . There are (C) 2 solutions. 

Problem 20. Suppose that on a parabola with vertex V and focus F there exists a point A such
that AF = 20 and AV = 21. What is the sum of all possible values of the length F V ?
40 41 43
(A) 13 (B) 3 (C) 3 (D) 14 (E) 3

Solution. Let the directrix be the x-axis, F = (0, 2d), V = (0, d), A = (x, y), and B = (0, y) for
some d > 0. By the definition of a parabola, y = 20. We compute x in two ways:

x2 = AF 2 − BF 2 = 202 − |20 − 2d|2


= AV 2 − BV 2 = 212 − |20 − d|2

Subtracting,

0 = 202 − 212 + (20 − d)2 − (20 − 2d)2


= 3d2 − 40d + 41.

40
The sum of all possible values of F V = d is (B) by Vieta’s. 
3

7
2021 AMC 12A Solution Manual

Problem 21. The five solutions to the equation

(z − 1)(z 2 + 2z + 4)(z 2 + 4z + 6) = 0

may be written in the form xk + yk i for 1 ≤ k ≤ 5, where xk and yk are real. Let E be the
unique ellipse that passes through the points (x1 , y1 ), (x2 , y2 ), (x3 , y3 ), (x4 , y4 ) and (x5 , y5 ). The
pm
eccentricity of E can be written in the form n , where m and n are relatively prime positive
integers. What is m + n? (Recall that the eccentrictiy of an ellipse E is the ratio ac , where 2a is
the length of the major axis of E and 2c is the distance between its two foci.)
(A) 7 (B) 9 (C) 11 (D) 13 (E) 15
√ √
Solution. The roots of the polynomial
√ are
√ z = 1, z = −1 ± i 3, and z = −2 ± i 2, hence the five
points on E are (1, 0), (−1, ± 3), (−2, ± 2).
By symmetry through the x-axis, the ellipse is of the form

E: a(x − r)2 + by 2 = 1.

We then have the relations

a(1 − r)2 = 1
a(1 + r)2 + 3b = 1
a(2 + r)2 + 2b = 1.

Eliminating b from the latter two,

1 = 3 a(2 + r)2 + 2b − 2 a(1 + r)2 + 3b = a r2 + 8r + 10 ,


    

hence
1 9
r2 + 8r + 10 = = (1 − r)2 =⇒ r = − .
a 10
100 120

it follows that a = 361 and b = 361 , so the eccentricity is 1/ 6. The requested sum is 1 + 6 =
(A) 7 

Problem 22. Suppose that the roots of the polynomial P (x) = x3 + ax2 + bx + c are cos 2π 4π
7 , cos 7 ,

and cos 7 , where angles are in radians. What is abc?

3
3 1 7 1 1
(A) − 49 (B) − 28 (C) 64 (D) 32 (E) 28

Solution. Recall

1 + e2πi/7 + · · · + e12πi/7 = 0 =⇒ e2πi/7 + e4πi/7 + e6πi/7 = −1/2,

and in particular cos 2π 4π 6π


7 + cos 7 + cos 7 = −1/2.
Note θ = 2π 4π 6π
7 , 7 , 7 are solutions to the equation cos θ + cos 2θ + cos 3θ = −1/2, so letting x = cos θ
implies that
1 2π 4π 6π
x + (2x2 − 1) + (4x3 − 3x) = − has roots x = cos , cos , cos .
2 7 7 7
1
Thus, the polynomial in question is x3 + 12 x2 − 21 x − 1
8 and the requested answer is (D) . 
32

8
2021 AMC 12A Solution Manual

Remark. Perhaps it is easier to motivate the solution as follows. Let t = e2πi/7 and x = t + t−1 =
2 cos 2π/7. Then x2 − 2 = t2 + t−2 and x3 − 3x = t3 + t−3 . Moreover, t6 + t5 + · · · + 1 = 0 implies
t3 + t2 + t + t−1 + t−2 + t−3 = −1, i.e.

x + (x2 − 2) + (x3 − 3x) has root 2 cos 2π/7.

It certainly seems logical that the Galois conjugates of 2 cos 2π 4π


7 would be 2 cos 7 and 2 cos 7

x
(especially given the phrasing of the problem), so simply replace x 7→ 2 to get the desired polynomial
form.
Let ω = e2iπ/7 . Note that
2π ω + ω −1
cos =
7 2
4π ω + ω −2
2
cos =
7 2
6π ω + ω −3
3
cos = .
7 2
Let these be r, s, t respectively. By Vieta’s formulas, note that the desired quantity is

(−rst)(rs + st + tr)(−r − s − t) = (r + s + t)(rs + st + tr)(rst).

Note that 1 + ω + · · · + ω 6 = 0. We have


ω + ω 2 + ω 3 + ω −1 + ω −2 + ω −3
r+s+t=
2
ω + ω2 + · · · + ω6
=
2
1
=− .
2
Then,

ω 3 + ω + ω −1 + ω −3 ω 5 + ω + ω −1 + ω −5 ω 4 + ω 2 + ω −2 + ω −4
rs + st + tr = + +
4 4 4
2(ω + ω 2 + · · · + ω 6 )
=
4
1
=− .
2
Finally,

ω 6 + ω 4 + ω 2 + ω 0 + ω 0 + ω −2 + ω −4 + ω −6
rst =
8
(ω + ω + · · · + ω ) + ω 0
6 5 0
=
8
1
= .
8

1
Multiplying yields the answer of (D)
32

9
2021 AMC 12A Solution Manual

Problem 23. Frieda the frog begins a sequence of hops on a 3 × 3 grid of squares, moving one
square on each hop and choosing at random the direction of each hop up, down, left, or right.
She does not hop diagonally. When the direction of a hop would take Frieda off the grid, she
”wraps around” and jumps to the opposite edge. For example if Frieda begins in the center square
and makes two hops ”up”, the first hop would place her in the top row middle square, and the
second hop would cause Frieda to jump to the opposite edge, landing in the bottom row middle
square. Suppose Frieda starts from the center square, makes at most four hops at random, and
stops hopping if she lands on a corner square. What is the probability that she reaches a corner
square on one of the four hops?
9 5 3 25 13
(A) 16 (B) 8 (C) 4 (D) 32 (E) 16

Solution. We complementary count, and determine the probability we never reach a corner square.
Denote by A the center, and B a square adjacent to the center. Then the first hop lands on B.

• If the second hop lands on A (with probability 14 ), then the third hop lands on B always, and
there is a 21 chance the fourth hop lands on a non-corner square. The probability in this case
is 41 · 12 = 81 .

• If the second hop lands on B, then there is a 41 chance the third hop lands on A, and a 14
chance the third hop lands on B. In the former subcase, the fourth hop always lands on a
non-corner square, and in the latter subcase, there is a 12 chance the fourth hop lands on a
non-corner square. The probability in this case is 41 · ( 14 + 41 · 12 ) = 32
3
.

The requested probability is


1 3 25
1− − = (D)
4 32 32


Problem 24. Semicircle Γ has diameter AB of length √ 14. Circle Ω lies◦ tangent to AB at a point
P and
√ intersects Γ at points Q and R. If QR = 3 3 and ∠QP R = 60 , then the area of 4P QR
a b
is c , where a and c are relatively prime positive integers, and b is a positive integer not divisible
by the square of any prime. What is a + b + c?
(A) 110 (B) 114 (C) 118 (D) 122 (E) 126
QR
Solution. First, by Extended Law of Sines, we have that the radius of Ω is 2 sin 60◦ = 3. Let M
be the midpoint of QR, O be the center of Γ, and X be the center of Ω. Since 4OQR is isosceles,
OM is perpendicular to QR. Thus we have that X lies on OM since it is the circumcenter of
4P QR. Computing lengths, we have OM = 13 2 by Pythagorean theorem on 4ORM and XM = 2
3
13 3
from isosceles triangle XQR. Thus OX = 2 − 2 = 5 and OP = 4 from Pythagorean theorem on
4OXP .
To find [P QR], we will find the height from P to QR. Let the foot of the perpendicular from X
to the P -altitude be D. Since P D k OM , we know that 4XDP ∼ 4OP X. This means that
P X = 35 OP = 95 . Now note that the bottom portion of the P -altitude after subtracting P D is
equal to XM , so the height of the triangle is 95 + 23 = 33
10 . The area is simply

1 33 √ 99 3
· ·3 3= =⇒ 99 + 3 + 20 = (D) 122
2 10 20

10
2021 AMC 12A Solution Manual

Problem 25. Let d(n) denote the number of positive integers that divide n, including 1 and n.
For example, d(1) = 1, d(2) = 2, and d(12) = 6. (This function is known as the divisor function.)
Let
d(n)
f (n) = √3
.
n
There is a unique positive integer N such that f (N ) > f (n) for all positive integers n 6= N . What
is the sum of the digits of N ?
(A) 5 (B) 6 (C) 7 (D) 8 (E) 9

Solution. Let n = i pei i , where p1 = 2, p2 = 3, etc. are the primes in increasing order and ei are
Q

nonnegative integers. Then Q


i (ei + 1)
Y ei + 1
f (n) = Q ei /3
= ei /3
.
p
i i i p i
(e +1) 3
It is equivalent to maximize (f (n))3 = i ipei . Thus, it remains to find the optimal ei for each i.
Q
i
We go term-by-term, noting that it is only necessary to check until the expression first decreases,
as exponentials increase more quickly than polynomials.

i ei ((ei + 1)3 )/pei i


1 0 13 /20 = 1
1 1 23 /21 = 4
1 2 3 /22 = 6.25
3

1 3 43 /23 = 8
1 4 53 /24 < 8
2 0 13 /30 = 1
2 1 23 /31 ≈ 2.67
2 2 33 /32 = 3
2 3 43 /33 < 3
3 0 13 /50 = 1
3 1 23 /51 = 1.6
3 2 33 /52 = 1.08
4 0 13 /70 = 1
4 1 23 /71 ≈ 1.14
4 2 33 /72 < 1

8
Note that we do not need to check p ≥ 11, as ei = 1 yields p < 1, which is suboptimal.
Thus, the answer is N = 23 · 32 · 5 · 7 = 2520, which has a digit sum of (E) 9
Remark: It is sufficient to stop at p = 3, for 32 | N leaves only one possibility for the digit sum of
N among the answer choices. 

11

You might also like